Is X uncountable if there exists a B subset of uncountable with X-B uncountable?

  • Thread starter Thread starter lreyvega
  • Start date Start date
  • Tags Tags
    Set
Click For Summary
SUMMARY

The discussion centers on proving that if a set X is uncountable, then there exists a subset B of X that is also uncountable, such that the difference X-B remains uncountable. The proof utilizes the axiom of choice to well-order the set X and constructs the subset B by excluding elements of the form b=a+1 for a in X. This leads to the conclusion that both B and X-B are uncountable, confirming the initial claim.

PREREQUISITES
  • Understanding of sigma-fields and their generation, specifically 2_{X} and singleton sets.
  • Familiarity with the concepts of countable and uncountable sets in set theory.
  • Knowledge of the axiom of choice and its implications in set theory.
  • Basic proof techniques in mathematical logic, particularly proof by contradiction.
NEXT STEPS
  • Study the properties of sigma-fields and their applications in measure theory.
  • Explore the implications of the axiom of choice in set theory and its role in constructing uncountable sets.
  • Learn about well-ordering principles and their use in proofs involving infinite sets.
  • Investigate further examples of countable vs. uncountable sets and their subsets.
USEFUL FOR

Mathematicians, particularly those focused on set theory, logic, and measure theory, as well as students tackling advanced topics in mathematics related to countability and uncountability.

lreyvega
Messages
1
Reaction score
0
Hi everyone! I have the following problem

Homework Statement



X is countable set if and only if the sigma-field 2_{X} is generated by the sets of the form \left\{\omega\right\} with \omega\in X


Homework Equations





The Attempt at a Solution



The easy part is X is countable then...
I'm trying to prove the other part. So, I know that the sigma-field 2_{X} is generated by the sets of the form \left\{\omega\right\} with \omega\in X. This means that 2_{X} can be written as

2_{X}=\cap \sigma_{\alpha}

where \sigma_{\alpha} are all the sigma-fields that contain the family of the singletons. But 2_{X} is the biggest sigma-field that can be constructed in X. That means that \sigma_{\alpha}=2_{X} \forall
\alpha.

Now suppose that X is uncountable. Be the sigma-field \sigma constructed in the following manner:

A\subseteq X is in the \sigma if A is countable or finite or if X\A is countable or finite.

It is clear that this sigma-field contains the class formed by the singletons in X. However, this class it is not 2_{X}, because if X is uncountable exists B\subset uncountable with X\B uncountable. That is, B is not in \sigma. This means that

2_{X} is not equal to \cap \sigma_{\alpha}

So, this is a contradiction and X is numerable.

So, my questions are:

1) Is this proof correct?
2) If it is correct, I think that the critical point in it is this fact I used:

If X is uncountable exists B\subset uncountable with X\B uncountable

How can I prove this? I think that I need to use the axiom of choice, but I have no clue how to do it.

Thanks and sorry for my bad english
 
Physics news on Phys.org
If X is uncountable exists B\subset uncountable with X\B uncountable

How can I prove this? I think that I need to use the axiom of choice, but I have no clue how to do it.

The rest of your proof looks good. To prove this step, well-order X (OK by axiom of choice).

Now let B be the set of points b in X that are not of the form b=a+1, for a in X.

Then B is uncountable (assume not and deduce a contradiction).

Then X-B is clearly uncountable (for each b in B, we have b+1 in X-B).
 
Question: A clock's minute hand has length 4 and its hour hand has length 3. What is the distance between the tips at the moment when it is increasing most rapidly?(Putnam Exam Question) Answer: Making assumption that both the hands moves at constant angular velocities, the answer is ## \sqrt{7} .## But don't you think this assumption is somewhat doubtful and wrong?

Similar threads

  • · Replies 8 ·
Replies
8
Views
2K
  • · Replies 3 ·
Replies
3
Views
3K
Replies
1
Views
2K
Replies
3
Views
2K
  • · Replies 1 ·
Replies
1
Views
3K
  • · Replies 15 ·
Replies
15
Views
3K
  • · Replies 6 ·
Replies
6
Views
2K
Replies
20
Views
4K
  • · Replies 2 ·
Replies
2
Views
2K
  • · Replies 2 ·
Replies
2
Views
1K